Vous êtes sur la page 1sur 11

COSMOS: Complete Online Solutions Manual Organization System

Chapter 3, Problem 1. Redraw the circuit of Fig. 3.42, consolidating nodes into the minimum number possible.

FIGURE 3.42

Chapter 3, Solution 1.

Chapter 3, Problem 6. (a) Determine the current labeled iz in the circuit shown in Fig. 3.45. (b) If the resistor carrying 3 A has a value of 1 W , what is the value of the resistor carrying -5 A?

FIGURE 3.45
Engineering Circuit Analysis, 7/e, W.H. Hayt, Jr., J.E. Kemmerly, and S.M. Durbin 2007 The McGraw-Hill Companies.

COSMOS: Complete Online Solutions Manual Organization System

Chapter 3, Solution 6. (a) By KCL at the bottom node: So iZ = 9 A. 2 3 + iZ 5 3 = 0

(b) If the left-most resistor has a value of 1 W, then 3 V appears across the parallel network (the + reference terminal being the bottom node) Thus, the value of the other resistor is given by R= 3 = 600 mW . -(-5)

Engineering Circuit Analysis, 7/e, W.H. Hayt, Jr., J.E. Kemmerly, and S.M. Durbin 2007 The McGraw-Hill Companies.

COSMOS: Complete Online Solutions Manual Organization System

Chapter 3, Problem 8. Referring to Fig. 3.47,

(c) Find iz if ix = iy = iz

(a) Find ix if iy = 2 A and iz = 0 A. (b) Find iy if ix = 2 A and iz = 2 iy .

FIGURE 3.47

Chapter 3, Solution 8. By KCL, we may write:

(b) iy = 3 + ix 5 iz iy = 2 + 2 2 iy

(a) ix = 2 + iy + iz = 2 + 2 + 0 = 4 A

5 + iy + iz = 3 + ix

Thus, we find that iy = 0. (c) 5 + iy + iz = 3 + ix

5 + ix + ix = 3 + ix so ix = 3 5 = -2A.

Engineering Circuit Analysis, 7/e, W.H. Hayt, Jr., J.E. Kemmerly, and S.M. Durbin 2007 The McGraw-Hill Companies.

COSMOS: Complete Online Solutions Manual Organization System

Chapter 3, Problem 15. Find R and G in the circuit of Fig. 3.51b if the 5 A source is supplying 100 W and the 40 V source is supplying 500 W.

FIGURE 3.51

Chapter 3, Solution 15. We begin by making use of the information given regarding the power generated by

the 5-A and the 40-V sources. The 5-A source supplies 100 W, so it must therefore have a terminal voltage of 20 V. The 40-V source supplies 500 W, so it must therefore provide a current IX of 12.5 A.

Engineering Circuit Analysis, 7/e, W.H. Hayt, Jr., J.E. Kemmerly, and S.M. Durbin 2007 The McGraw-Hill Companies.

COSMOS: Complete Online Solutions Manual Organization System

(2) By KVL, -VG (-110) + 40 = 0 Now that we know the voltage across the unknown conductance G, we need only to find the current flowing through it to find its value by making use of Ohms law. So VG = 150 V

Thus, R = 34 W.

(1) By KVL, 40 + (110) + R(5) 20 = 0

KCL provides us with the means to find this current: The current flowing into the + terminal of the 110-V source is 12.5 + 6 = 18.5 A. By Ohms law, Ix = G VG So G = 13.5/ 150 or Then, Ix = 18.5 5 = 13.5 A

G = 90 mS

Chapter 3, Problem 20. Use Ohms and Kirchhoff s laws on the circuit of Fig. 3.56 to find (a) vx ; (b) iin; (c) Is ; (d) the power provided by the dependent source.
Engineering Circuit Analysis, 7/e, W.H. Hayt, Jr., J.E. Kemmerly, and S.M. Durbin 2007 The McGraw-Hill Companies.

COSMOS: Complete Online Solutions Manual Organization System

FIGURE 3.56

Chapter 3, Solution 20. (a) By KVL, -2 + vx + 8 = 0 so that (b) By KCL at the top left node, iin = 1 + IS + vx/4 6 or iin = 23 A vx = -6 V.

(c) By KCL at the top right node, IS + 4 vx = 4 - vx/4 So IS = 29.5 A.

Engineering Circuit Analysis, 7/e, W.H. Hayt, Jr., J.E. Kemmerly, and S.M. Durbin 2007 The McGraw-Hill Companies.

(d) The power provided by the dependent source is 8(4vx) = -192 W.

COSMOS: Complete Online Solutions Manual Organization System

Chapter 3, Problem 38. Find the power absorbed by element X in the circuit of Fig. 3.71 if it is a (a) 4 k W resistor; (b) 20 mA independent current source, reference arrow downward; (c) dependent current source, reference arrow downward, labeled 2ix ; (d) 60 V independent voltage source, + reference at top.

FIGURE 3.71 Chapter 3, Problem 38. Find the power absorbed by element X in the circuit of Fig. 3.71 if it is a (a) 4 k W resistor; (b) 20 mA independent current source, reference arrow downward; (c) dependent current source, reference arrow downward, labeled 2ix ; (d) 60 V independent voltage source, + reference at top.

FIGURE 3.71

Chapter 3, Problem 47. Compute the current labeled i in each of the circuits in Fig. 3.79.

Engineering Circuit Analysis, 7/e, W.H. Hayt, Jr., J.E. Kemmerly, and S.M. Durbin 2007 The McGraw-Hill Companies.

COSMOS: Complete Online Solutions Manual Organization System

FIGURE 3.79

Chapter 3, Solution 47. (a) Combine the 12-V and 2-V series connected sources to obtain a new 12 2 = 10 V source, with the + reference terminal at the top. The result is two 10-V sources in parallel, which is permitted by KVL. Therefore, i = 10/1000 = 10 mA. (b) No current flows through the 6-V source, so we may neglect it for this calculation. The 12-V, 10-V and 3-V sources are connected in series as a result, so we replace them with a 12 + 10 3 = 19 V source as shown

Thus,

i = 19/5 = 3.8 A.

Chapter 3, Problem 50. In the circuit of Fig. 3.82, choose v1 to obtain a current ix of 2 A.

FIGURE 3.82

Engineering Circuit Analysis, 7/e, W.H. Hayt, Jr., J.E. Kemmerly, and S.M. Durbin 2007 The McGraw-Hill Companies.

COSMOS: Complete Online Solutions Manual Organization System

Chapter 3, Solution 50. We first simplify as shown, making use of the fact that we are told ix = 2 A to find the voltage across the middle and right-most 1-W resistors as labeled.

By KVL, then, we find that

v1 = 2 + 3 = 5 V.

Engineering Circuit Analysis, 7/e, W.H. Hayt, Jr., J.E. Kemmerly, and S.M. Durbin 2007 The McGraw-Hill Companies.

Vous aimerez peut-être aussi